LSAT and Law School Admissions Forum

Get expert LSAT preparation and law school admissions advice from PowerScore Test Preparation.

 Administrator
PowerScore Staff
  • PowerScore Staff
  • Posts: 8916
  • Joined: Feb 02, 2011
|
#23041
Complete Question Explanation

Assumption-SN. The correct answer choice is (A)

The stimulus concerns the attempt of pet-lovers to change the rules of a residential building. The conditional for changing the rules of a residential building is:
  • "Rules Change → 10% Petition AND ONLY THEN Majority Vote."

    [Technically correct: "Rules Change → Majority Vote → 10% Petition."]
The stimulus concludes that since the rules were not changed, the pet lovers were voted down by a majority of the tenants.

In order to be defeated by a majority vote, however, the pet-lovers would first have had to obtain the required 10% of tenant signatures on their petition. Since you are asked to identify an assumption in the argument, you should focus on the fact that the argument neglects the possibility that the pet-lovers never got the required signatures, so a vote never occurred.

Answer choice (A): This is the correct answer choice. For the argument to make a conclusion about the outcome of a vote, the argument needs to assume that the vote was possible in the first place. Without 10% of tenant signatures, the vote would not have ever occurred.

Answer choice (B): The argument would definitely not presume that less than 10% of tenants signed the petition, because that would eliminate the possibility that a vote occurred. This choice is contrary to the argument, and incorrect.

Answer choice (C): Since only a simple majority seems required to vote down the proposal, the stimulus does not need to assume anything so drastic as the idea that 90% of tenants opposed changing the rules.

Answer choice (D): The stimulus would clearly not assume that 10% support is sufficient to ensure the adoption of a rule change, because the stimulus states that majority support is required. This choice is contrary to statements in the stimulus, and thus should be immediately eliminated.

Answer choice (E): The idea that a failure to obtain 10% of tenant signatures would ensure that a proposal would be voted down by a majority of tenants is immediately incompatible with the argument, which states that without 10% of tenant signatures, the proposal will never even be voted upon. This choice is somewhat contradictory to the stimulus, and is thus incorrect.
 NghiemB
  • Posts: 7
  • Joined: Aug 06, 2011
|
#1503
For this problem, I selected B as my answer. However, the correct answer was A. The conclusion states that the pet lovers were voted down on their proposal. I assumed that since their proposal was voted down, this means that the pet lovers did not obtain at least 10 % tenant's signature, and thus did not qualified to be put to a majority vote.
However, the answer was A. The pet lovers succeeded in obtaining the signature of 10 percent of the tenants their petition.
If they succeeded in getting the signature, wouldn't it mean that their proposal will be put to a majority vote? I know that even if their proposal were put to vote does not necessarily mean that the rule regarding pets will be changed. But since the conclusion states that their proposal were voted down, does it not follow that in order to be voted down, they had to get the 10 percent to have the proposal put to a majority vote? Confusing ! . Please let me know.
User avatar
 Dave Killoran
PowerScore Staff
  • PowerScore Staff
  • Posts: 5853
  • Joined: Mar 25, 2011
|
#1504
I'm not certain, but I think maybe you are mis-reading part of the question. The stimulus states that the following relationship is in effect; the "only if" creates the following relationship:

Premise: Put to majority vote → obtain signatures of 10%

Conclusion: Lost majority vote

So, if the proposal lost a majority vote as stated in the conclusion, then according to the premise, in order for the vote to occur, the group must have obtained the signatures of 10%. Because that is not stated in the argument, it is assumed, and answer choice (A) is correct.

Note that "put to a majority vote" just means that the proposal goes up for a vote; it doesn't mean that the won the vote.

Please let me know if that helps. Thanks!

Get the most out of your LSAT Prep Plus subscription.

Analyze and track your performance with our Testing and Analytics Package.